Q5

 
Rachel.Jay.Shapiro
Thanks Received: 0
Vinny Gambini
Vinny Gambini
 
Posts: 3
Joined: December 02nd, 2009
 
 
 

Q5

by Rachel.Jay.Shapiro Tue Mar 09, 2010 10:48 pm

Hi,
I'm having trouble with the last question in this game (#5). While I believe that my set-up for the game was correct because the preceding questions were fine, I feel as though I'm missing certain inferences that allow this question to be solved without trial and error. Because of this, I was unable to finish. Please help.
Thanks!
User avatar
 
ManhattanPrepLSAT1
Thanks Received: 1909
Atticus Finch
Atticus Finch
 
Posts: 2851
Joined: October 07th, 2009
 
This post thanked 1 time.
 
 

Re: PT43, S4, G1 - There are exactly six groups in this year's

by ManhattanPrepLSAT1 Tue Mar 09, 2010 11:17 pm

For question 5, plug and chug is the most likely way one would end up at the correct answer choice (B). Perhaps you have some work from a previous question that would allow you to see quickly that F can march immediately behind G. If not, you can create a quick hypothetical.

Here's a slide to help!
Attachments
PT43, S4, G1, Q5 Civic Parade.jpg
(176.12 KiB) Downloaded 461 times
 
nevenaashminova
Thanks Received: 0
Vinny Gambini
Vinny Gambini
 
Posts: 1
Joined: May 23rd, 2013
 
 
 

Re: Q5

by nevenaashminova Wed Jun 19, 2013 9:44 pm

I think Q5 was the hardest one for this game. For me the biggest worry was - Am I looking for a group that can NEVER be behind G? Or is there a particular frame I should focus on? Just when G is 1st? 3rd? or 5th?
Another thing I thought about was - Well J is a floater it should have the least amount of trouble getting behind G. Apparently not :roll: Very counterintuitive.
User avatar
 
tommywallach
Thanks Received: 468
Atticus Finch
Atticus Finch
 
Posts: 1041
Joined: August 11th, 2009
 
 
 

Re: Q5

by tommywallach Sun Jun 23, 2013 12:37 pm

Hey Nevena,

This is simply a very tough question. If you've made a lot of frames, it becomes relatively easy (the frame where G is first allows you to cross out A and D, etc.), but it's still a chugfest. Just keep good track of all of your "successful" combinations as you work through the other questions. You could probably get this down to 50/50 just by doing that, then use your frames to eliminate the one wrong answer that's left.

I wish there were an easier way.

Be careful though. "Never behind G" implies that it would definitively be true in ANY frame, so there shouldn't be any thought about trying to find the frame where it can't be behind G. You should be trying to find frames where things CAN be behind G, and eliminating as you do so.

Finally, your guess about the floatiness of J was a good one, it just turned out to be wrong. I'd still call it the best guess, if you didn't have any time to do anything, but even the best guess isn't going to be right more than 50 or 60% of the time.

Good luck!

-t
Tommy Wallach
Manhattan LSAT Instructor
twallach@manhattanprep.com
Image